跳到主要内容
正文中添加了618个字符
来源 链接
克劳德·莱博维奇
  • 26.5万
  • 52
  • 116
  • 253

我们有$$F_n=\frac{\phi^n-(-\frac{1}{\phi})^n}{\sqrt{5}}$$什么可以颠倒书写$$\斐^{2n}-F_n\sqrt{5}\,\phi^n-(-1)^n=0$$它是平方英寸$\菲^n$.所以$$\phi^n=\frac{F_n\sqrt{5}+\sqrt}5F_n^2\pm4}}{2}\表示n\log$$这个$\下午4点$几乎不雅观。$$\phi^n=\frac{F_n\sqrt{5}+\sqrt}5F_n^2\pm4}}{2}\暗示n_pm\log(\phi)=\log\Bigg[\frac}F_n\scrt{5}+\sqrt[2\pm4}{2{2}\Bigg]$$两种解决方案之一是整数$$\剩余(\开始{数组}{cccc}n&F_n&n_-&n_+\\3 & 2 & 3.000000000 & 3.209573980 \\4和3&3.907487979&4.000000000\\5 & 5 & 5.000000000 & 5.033256487 \\6 & 8 & 5.987011534 & 6.000000000 \\7 & 13 & 7.000000000 & 7.004918572 \\8 & 21 & 7.998115113 & 8.000000000 \\9 & 34 & 9.000000000 & 9.000719061 \\10 & 55 & 9.999725212 & 10.00000000\结束{数组}\右侧)$$

尝试如果不是这样,那么这个数字就不是斐波那契。尝试对于F_n美元=144$然后$12345678987654321$,这个两种解决方案是结果是$n_pm=78.669724$; 所以,它不是斐波那契数(但我们知道最接近的一个)。 $$n=11.999960\qquad\text{和}\qquad n=12$$$$F_{78}=8944394323791464\quad<\quad 1234567898765431$$ $$F_{79}=14472334024676221\quad>\quad 12345678987654321$$

编辑

考虑两个根之间的区别:考虑$F_n$做大做系列扩张$$n_+-n_-\sim\frac{2}{5F_n^2\log(\phi)}<\frac{1}{F_n^2}$$

我们有$$F_n=\frac{\phi^n-(-\frac{1}{\phi})^n}{\sqrt{5}}$$什么可以颠倒书写$$\斐^{2n}-F_n\sqrt{5}\,\phi^n-(-1)^n=0$$这是一个平方美元\phi^n$.所以$$\phi^n=\frac{F_n\sqrt{5}+\sqrt}5F_n^2\pm4}}{2}\表示n\log$$这个$\下午4点$几乎不雅观。

尝试对于F_n美元=144$然后这个两种解决方案是 $$n=11.999960\qquad\text{和}\qquad n=12$$

编辑

考虑两个根之间的区别:考虑$F_n$做大并进行系列扩展$$n_+-n_-\sim\frac{2}{5F_n^2\log(\phi)}<\frac{1}{F_n^2}$$

我们有$$F_n=\frac{\phi^n-(-\frac{1}{\phi})^n}{\sqrt{5}}$$什么可以颠倒书写$$\斐^{2n}-F_n\sqrt{5}\,\phi^n-(-1)^n=0$$它是平方英寸美元\phi^n$.所以$$\phi^n=\frac{F_n\sqrt{5}+\sqrt}5F_n^2\pm4}}{2}\暗示n_pm\log(\phi)=\log\Bigg[\frac}F_n\scrt{5}+\sqrt[2\pm4}{2{2}\Bigg]$$两种解决方案之一是整数$$\剩余(\开始{数组}{cccc}n&F_n&n_-&n_+\\3 & 2 & 3.000000000 & 3.209573980 \\4 & 3 & 3.907487979 & 4.000000000 \\5 & 5 & 5.000000000 & 5.033256487 \\6 & 8 & 5.987011534 & 6.000000000 \\7 & 13 & 7.000000000 & 7.004918572 \\8 & 21 & 7.998115113 & 8.000000000 \\9 & 34 & 9.000000000 & 9.000719061 \\10 & 55 & 9.999725212 & 10.00000000\结束{数组}\右侧)$$

如果不是这样,那么这个数字就不是斐波那契。尝试对于$12345678987654321$,这个结果是$n_pm=78.669724$; 所以,它不是斐波那契数(但我们知道最接近的一个)。 $$F_{78}=8944394323791464\quad<\quad 1234567898765431$$ $$F_{79}=14472334024676221\quad>\quad 12345678987654321$$

编辑

考虑两个根之间的区别:考虑$F_n$做大做系列扩张$$n_+-n_-\sim\frac{2}{5F_n^2\log(\phi)}<\frac{1}{F_n^2}$$

正文中添加了1个字符
来源 链接
克劳德·莱博维奇
  • 26.5万
  • 52
  • 116
  • 253

我们有$$F_n=\frac{\phi^n-(-\frac{1}{\phi})^n}{\sqrt{5}}$$什么可以反转书写$$\斐^{2n}-F_n\sqrt{5}\,\phi^n-(-1)^n=0$$这是一个卡塔尔语二次的在里面美元\phi^n$.所以$$\phi^n=\frac{F_n\sqrt{5}+\sqrt}5F_n^2\pm4}}{2}\表示n\log$$这个$\下午4点$几乎不雅观。

尝试F_n美元=144$那么这两个解决方案是$$n=11.999960\qquad\text{和}\qquad n=12$$

编辑

考虑一下根:考虑$F_n$做大做系列扩张$$n_+-n_-\sim\frac{2}{5F_n^2\log(\phi)}<\frac{1}{F_n^2}$$

我们有$$F_n=\frac{\phi^n-(-\frac{1}{\phi})^n}{\sqrt{5}}$$什么可以颠倒书写$$\斐^{2n}-F_n\sqrt{5}\,\phi^n-(-1)^n=0$$这是一个卡塔尔语在里面$\菲^n$.所以$$\phi^n=\frac{F_n\sqrt{5}+\sqrt}5F_n^2\pm4}}{2}\表示n\log$$这个$\下午4点$几乎不雅观。

尝试F_n美元=144$那么这两个解决方案是$$n=11.999960\qquad\text{和}\qquad n=12$$

编辑

考虑一下根:考虑$F_n$做大做系列扩张$$n_+-n_-\sim\frac{2}{5 F_n^2 \log(\phi)}<\frac{1}{F_n^2}$$

我们有$$F_n=\frac{\phi^n-(-\frac{1}{\phi})^n}{\sqrt{5}}$$什么可以颠倒书写$$\斐^{2n}-F_n\sqrt{5}\,\phi^n-(-1)^n=0$$这是一个二次的在里面美元\phi^n$.所以$$\phi^n=\frac{F_n\sqrt{5}+\sqrt}5F_n^2\pm4}}{2}\表示n\log$$这个$\下午4点$几乎不雅观。

尝试F_n美元=144$那么这两个解决方案是$$n=11.999960\qquad\text{和}\qquad n=12$$

编辑

考虑一下根:考虑$F_n$做大做系列扩张$$n_+-n_-\sim\frac{2}{5F_n^2\log(\phi)}<\frac{1}{F_n^2}$$

正文中添加了192个字符
来源 链接
克劳德·莱博维奇
  • 26.5万
  • 52
  • 116
  • 253

我们有$$F_n=\frac{\phi^n-(-\frac{1}{\phi})^n}{\sqrt{5}}$$什么可以颠倒书写$$\斐^{2n}-F_n\sqrt{5}\,\ phi^n-(-1)^n=0$$在中是qadratic美元\phi^n$.所以$$\phi^n=\frac{F_n\sqrt{5}+\sqrt}5F_n^2\pm4}}{2}\表示n\log$$这个$\下午4点$几乎不雅观。

尝试F_n美元=144$那么这两个解决方案是$$n=11.999960\qquad\text{和}\qquad n=12$$

编辑

考虑两个根之间的区别:考虑$F_n$做大做系列扩张$$n_+-n_-\sim\frac{2}{5F_n^2\log(\phi)}<\frac{1}{F_n^2}$$

我们有$$F_n=\frac{\phi^n-(-\frac{1}{\phi})^n}{\sqrt{5}}$$什么可以反转书写$$\斐^{2n}-F_n\sqrt{5}\,\phi^n-(-1)^n=0$$在中是qadratic美元\phi^n$.所以$$\phi^n=\frac{F_n\sqrt{5}+\sqrt}5F_n^2\pm4}}{2}\表示n\log$$这个$\下午4点$几乎不雅观。

尝试$F_n=144美元$那么这两个解决方案是$$n=11.999960\qquad\text{和}\qquad n=12$$

我们有$$F_n=\frac{\phi^n-(-\frac{1}{\phi})^n}{\sqrt{5}}$$什么可以颠倒书写$$\斐^{2n}-F_n\sqrt{5}\,\phi^n-(-1)^n=0$$在中是qadratic美元\phi^n$.所以$$\phi^n=\frac{F_n\sqrt{5}+\sqrt}5F_n^2\pm4}}{2}\表示n\log$$这个$\下午4点$几乎不雅观。

尝试F_n美元=144$那么这两个解决方案是$$n=11.999960\qquad\text{和}\qquad n=12$$

编辑

考虑两个根之间的区别:考虑$F_n$做大做系列扩张$$n_+-n_-\sim\frac{2}{5F_n^2\log(\phi)}<\frac{1}{F_n^2}$$

来源 链接
克劳德·莱博维奇
  • 26.5万
  • 52
  • 116
  • 253
加载